The order of an element of a direct product of groups

Click For Summary
The discussion focuses on proving that the order of an element in the direct product of two finite groups, A and B, is the least common multiple of the orders of the individual elements a and b. It establishes that if (a,1)^n = (1,1) and (1,b)^n = (1,1), then both a^n = 1 and b^n = 1 must hold. Consequently, it is concluded that n divides the order of both a and b, leading to the assertion that the least common multiple of their orders must equal n. Participants clarify the reasoning behind the implications of a^n = 1 on the divisibility of n by the orders of a and b. The conversation emphasizes the importance of understanding the relationship between the orders of the elements and their implications in group theory.
Mr Davis 97
Messages
1,461
Reaction score
44

Homework Statement


Let ##A## and ##B## be finite groups, and ##A \times B## be their direct product. Given that ##(a,1)## and ##(1,b)## commute, and that ##(a,1)^n = (a^n,1)## and ##(1,b)^n = (1,b^n)## for all a and b, show that the order of ##(a,b)## is the least common multiple of the orders of a and b.

Homework Equations

The Attempt at a Solution


Let ##n## be the order of ##(a,b)##. Then ##(a,b)^n = (1,1)##,
##[(a,1)(1,b))]^n = (1,1)##
##(a,1)^n (1,b)^n = (1,1)##
##(a^n,1)(1,b^n) = (1,1)##
##(a^n,b^n) = (1,1)##
So ##a^n = 1##and ##b^n = 1##. It must be the case that n divides the order of ##a## and the order of ##a##. The smallest number that divides both is by definition the least common multiple.

Is this proof okay?
 
Physics news on Phys.org
Almost.
Mr Davis 97 said:
So ##a^n = 1## and ##b^n = 1##.
O.k. until here.
It must be the case that n divides the order of ##a## and the order of ##a##. The smallest number that divides both is by definition the least common multiple.
It must be the case that the order of ##a## and the order of ##b## divide ##n##. The smallest number that is divided by both is by definition the least common multiple.

Do you know why ##a^{ord(a)}=1=a^n## implies ##n\,\vert \,ord(a)\,##?
 
fresh_42 said:
Almost.

O.k. until here.

It must be the case that the order of ##a## and the order of ##b## divide ##n##. The smallest number that is divided by both is by definition the least common multiple.

Do you know why ##a^{ord(a)}=1=a^n## implies ##n\,\vert \,ord(a)\,##?
Right, my brain slipped up when I was typing. I know that if ##a^n = 1##, then the order of a must divide ##n##, specifically because the division algorithm
 
fresh_42 said:
Almost.

O.k. until here.

It must be the case that the order of ##a## and the order of ##b## divide ##n##. The smallest number that is divided by both is by definition the least common multiple.

Do you know why ##a^{ord(a)}=1=a^n## implies ##n\,\vert \,ord(a)\,##?
Right, my brain slipped up when I was typing. I know that if ##a^n = 1##, then the order of a must divide ##n##, specifically because the division algorithm
 
  • Like
Likes fresh_42
Indeed, you cannot have nonzero remainder, for it would contradict the minimality of the order, therefore ##\mbox{ord}(a)\mid n ##, I'm guessing fresh_42 meant it this way. The other way need not hold.
 
Mr Davis 97 said:
Right, my brain slipped up when I was typing.
nuuskur said:
Indeed, you cannot have nonzero remainder, for it would contradict the minimality of the order, therefore ##\mbox{ord}(a)\mid n ##, I'm guessing fresh_42 meant it this way. The other way need not hold.
Oops. Slippery ground, indeed.
 
Question: A clock's minute hand has length 4 and its hour hand has length 3. What is the distance between the tips at the moment when it is increasing most rapidly?(Putnam Exam Question) Answer: Making assumption that both the hands moves at constant angular velocities, the answer is ## \sqrt{7} .## But don't you think this assumption is somewhat doubtful and wrong?

Similar threads

Replies
1
Views
2K
Replies
4
Views
2K
  • · Replies 4 ·
Replies
4
Views
1K
  • · Replies 1 ·
Replies
1
Views
2K
Replies
3
Views
1K
Replies
3
Views
2K
  • · Replies 16 ·
Replies
16
Views
5K
  • · Replies 1 ·
Replies
1
Views
2K
  • · Replies 1 ·
Replies
1
Views
2K
Replies
2
Views
1K